Search found 385 matches


So we can make n + n(n-4) = n^2 - 3n = n(n-3) triangles using either one or two edges, and our probability is: Hi Ian, can you please point out the flaw in my calculation for the numerator. Atleast 1 side of the polygon can be selected in n ways for n sided polygon...This gives you the 2 vertices o...

by debmalya_dutta

Fri Sep 24, 2010 12:51 pm
Forum: Problem Solving
Topic: Probability
Replies: 5
Views: 1595

Content-wise , both e-Gmat and Manhattan SC guide are the same. However , I would vote for e-Gmat because the way the content is presented , spoken about etc , it's easier to grasp and apply . Also , the exercises are good. Surely , I would suggest that you subscribe to e-gmat

by debmalya_dutta

Wed Sep 22, 2010 6:55 pm
Forum: I just Beat The GMAT!
Topic: My H Experience
Replies: 17
Views: 4640
by debmalya_dutta

Wed Sep 22, 2010 10:47 am
Forum: Data Sufficiency
Topic: DS - Aboslute Values
Replies: 2
Views: 920

Hey Gurpinder.... Like you already mentioned..Pick up the good stuff from this experience and move on to the next phase - re-attempt ... 35 on Verbal is good... Improving on maths is an easier task than improving on verbal because they are only a certain number of principles that are tested and I th...

by debmalya_dutta

Tue Sep 21, 2010 9:05 pm
Forum: I just Beat The GMAT!
Topic: My H Experience
Replies: 17
Views: 4640

DS - Aboslute Values

source MGMAT CAT
Is there a quick way to solve this ?

If a and b are integers, and |a| > |b|, is a · |b| < a – b?

(1) a < 0

(2) ab is greater than or equal to 0

Has anyone come across any quick approach to this problem (not the one in the provided by MGMAT)

by debmalya_dutta

Tue Sep 21, 2010 8:11 pm
Forum: Data Sufficiency
Topic: DS - Aboslute Values
Replies: 2
Views: 920

Hi GMAT Hacker, Congratulations on your great score! Kaplan is proud of the accuracy of our CATs, and all things being equal, you can expect a similar score on the real thing. That being said, all isn't equal. Stress, lack of endurance, unfamiliarity with the testing center, and rare oddball questi...

by debmalya_dutta

Tue Sep 21, 2010 7:07 am
Forum: Ask a Kaplan representative
Topic: Kaplan Practice Test
Replies: 3
Views: 2310

If x, y, and k are positive numbers such that (10x/(x+y)) + (20y/(x+y)) = k and if x < y, which of the following could be the vale of k? a. 10; b.12; c.15; d.18; e.30 Refer this link.. Had some good discussion just a couple of days back http://www.beatthegmat.com/quants-questions-t66380.html#298936

by debmalya_dutta

Mon Sep 20, 2010 8:25 pm
Forum: Problem Solving
Topic: Integer
Replies: 3
Views: 1247

If p is the product of the integers from 1 to 30, inclusive, what is the greatest integer k for which 3^k is a factor of p? a. 10; b.12; c.14; d.16; e.18 this is a tedious one ... p = 1.2.3.4.5.6..............................30 which includes are multiples of 3 starting with 3 and ending in 30 Conc...

by debmalya_dutta

Mon Sep 20, 2010 8:20 pm
Forum: Problem Solving
Topic: Integer
Replies: 3
Views: 1247

If n is a positive integer, what is the remainder when (3^(8n+3)) + 2 is divided by 5? a. 0; b.1; c. 2; d.3; e.4 let us find the unit's digit of 3^(8n+3) let's simplify it by putting n=1 so , we have 3 ^ 11 ... the units digit of 3 ^ 11 is 7 ..... Now back to 3^(8n+3)) + 2 ... we know the units dig...

by debmalya_dutta

Mon Sep 20, 2010 8:15 pm
Forum: Problem Solving
Topic: Integer
Replies: 3
Views: 1247

If vmt ≠ 0, is v^2*m^3*t^-4 > 0? 1. m > v^2 2. m > t^-4 Please explain this question, thanks![/list][/spoiler][/list] think the answer is d here In the question ... you really need to prove that m is positive because v^2 and t^-4 are always positive statement 1 : m > v^2 ..this means that m is po...

by debmalya_dutta

Mon Sep 20, 2010 6:01 pm
Forum: Data Sufficiency
Topic: Need help
Replies: 2
Views: 904

10) If p is positive integer, what is remainder when p is divided by 4? a. When P is divided by 8 remainder is 5 b. P is the sum of the squares of two consecutive positive integers. [spoiler]oa: (d)[/spoiler] how is 2 sufficient? Look at it this way .. it is mentioned that you have squares of 2 con...

by debmalya_dutta

Mon Sep 20, 2010 5:52 pm
Forum: Data Sufficiency
Topic: remainder question
Replies: 5
Views: 1475

2. The number of stamps Kaye and Alberto had were in the ration 5:3 respectively . After Kaye gave Alberto 10 of her stamps , the ratio of the number of stamps Kaye had to number Alberto had was 7:5. As a result of this gift, Kaye had how many more stamps than Alberto? A. 20 B. 30 C. 40 D. 60 E. 90...

by debmalya_dutta

Mon Sep 20, 2010 1:50 pm
Forum: Problem Solving
Topic: GMATPrep 1 Questions
Replies: 6
Views: 1467

1. For every positive integer n, the function h(n) is defined to be product of all even integers form 2 to n, inclusive. If p is the smallest prime factor of h(100)+1, then p is A. between 2 and 10 B. between 10 and 20 C. between 20 and 30 D. between 30 and 40 E. greater than 40 this one is E h(100...

by debmalya_dutta

Mon Sep 20, 2010 1:44 pm
Forum: Problem Solving
Topic: GMATPrep 1 Questions
Replies: 6
Views: 1467

Oa is B @deb...the conclusion is about those who drive less pay for the insurance of those who drive more... Now if the insurance comp. does not spend more on those who drive less then only people driving less are paying for the cost of those who drive more... lets say those who drive less pay 100 ...

by debmalya_dutta

Sun Sep 19, 2010 7:59 am
Forum: Critical Reasoning
Topic: Reasoning??
Replies: 6
Views: 1638

http://s3.postimage.org/vt3C9.jpg Statement 1: The length < 12 Given one side is 5 If the diagnol were to be 13 , then by pythogoras theorem , X = 12 But the statement 1 says that X< 12 ...hence the diagnol will surely be lesser than 13 So sufficient Statement 2 : Says diagnol is > 10 Take x= 12 .....

by debmalya_dutta

Sat Sep 18, 2010 11:01 am
Forum: Data Sufficiency
Topic: geometry
Replies: 4
Views: 1292